場の量子論⑤ マクスウェル方程式のラグランジアン密度 このエントリーをはてなブックマークに追加

ベクトル場のラグランジアンを確認する

マクスウェル方程式を導くラグランジアン密度

マクスウェル方程式を導くラグランジアン密度は以下のようにあらわせます。

\begin{align*} \mathcal{L}&=-\dfrac{1}{4\mu}F_{\mu\nu}F^{\mu\nu}-j_\mu A^\mu \\ F_{\mu\nu}&=\partial_\mu A_\nu-\partial_\nu A_\mu \end{align*}
さて、このことを以下で確かめてみます。ベクトルポテンシャル・スカラーポテンシャルの紹介記事で導いた方程式は以下のような式でした。$c=1/\sqrt{\mu\varepsilon}$を用いて、$c$を使えるところでは書き換えています。
\begin{align} \dfrac{\partial}{\partial t}\left(\nabla\cdot\boldsymbol{A}\right)+\nabla^2\phi&=-\dfrac{\rho}{\varepsilon} \label{eq:1}\\ \left(\nabla^2-\dfrac{1}{c^2}\dfrac{\partial^2}{\partial t^2}\right)\boldsymbol{A}-\nabla\left(\nabla\cdot\boldsymbol{A}+\dfrac{1}{c^2}\dfrac{\partial\phi}{\partial t}\right)&=-\mu\boldsymbol{j} \label{eq:2} \end{align}
いまは相対論的な話をしているのでこの時空を別々に扱うのはよくないでしょう。\eqref{eq:1}を$c$で割って、あえて余計な項を付け加えます。$c=1/\sqrt{\mu\varepsilon}$を利用すれば、
\begin{align} \left(\nabla^2-\dfrac{1}{c^2}\dfrac{\partial^2}{\partial t^2}\right)\dfrac{\phi}{c}+\dfrac{1}{c}\dfrac{\partial}{\partial t}\left(\nabla\cdot\boldsymbol{A}+\dfrac{1}{c^2}\dfrac{\partial\phi}{\partial t}\right)=-\mu\rho c \label{eq:3} \end{align}
\eqref{eq:2},\eqref{eq:3}をさらに見やすい形にして並べると、
\begin{align} \left(\nabla^2-\dfrac{1}{c^2}\dfrac{\partial^2}{\partial t^2}\right)\boldsymbol{A}-\nabla\left\{\nabla\cdot\boldsymbol{A}+\dfrac{1}{c}\dfrac{\partial}{\partial t}\left(\dfrac{\phi}{c}\right)\right\}&=-\mu\boldsymbol{j} \\ \left(\nabla^2-\dfrac{1}{c^2}\dfrac{\partial^2}{\partial t^2}\right)\dfrac{\phi}{c}+\dfrac{1}{c}\dfrac{\partial}{\partial t}\left\{\nabla\cdot\boldsymbol{A}+\dfrac{1}{c}\dfrac{\partial}{\partial t}\left(\dfrac{\phi}{c}\right)\right\}&=-\mu\rho c \end{align}
というわけで、これらの式にあうように以下のように4元物理量を定めます。
\begin{align*} A^\mu&=\left(\dfrac{\phi}{c},\boldsymbol{A}\right) \\ j^\mu&=\left(\rho c, \boldsymbol{j}\right) \\ \end{align*}
とすれば、以下のようになります。
\begin{align*} -\partial_\mu\partial^\mu A^\nu+\partial^\nu \partial_\mu A^\mu=-\mu j^\nu \end{align*}
さて、偏微分が入れ替えられることを考慮して、
\begin{align*} \partial_\mu(\partial^\mu A^\nu-\partial^\nu A^\mu)=\mu j^\nu \end{align*}
つまり、
\begin{align*} \partial_\mu F^{\mu\nu}=\mu j^\nu \end{align*}
となり、相対論的Maxwell方程式が導けました。

オイラー・ラグランジュ方程式から導出

さて、最初に示したラグランジアンからMaxwell方程式が導けることを示しましょう。スカラー場ほど簡単ではないです。
\begin{align*} \mathcal{L}&=-\dfrac{1}{4\mu}F_{\mu\nu}F^{\mu\nu}-j_\mu A^\mu \\ 0&=\dfrac{\partial\mathcal{L}}{\partial A^\alpha}-\partial_\beta\left(\dfrac{\partial \mathcal{L}}{\partial(\partial_\beta A^\alpha)}\right) \end{align*}
添え字の上げ下げによって、以下の式が成り立ちます。
\begin{align*} \dfrac{\partial \mathcal{L}}{\partial A^\alpha}(A_\lambda) &=\dfrac{\partial \mathcal{L}}{\partial A^\alpha}(\eta_{\lambda\rho}A^\rho) \\ &=\eta_{\lambda\rho}\delta^\rho_\alpha \\ &=\eta_{\lambda\alpha} \end{align*}
これを利用すると、
\begin{align*} \dfrac{\partial \mathcal{L}}{\partial A^\alpha}=-j_\mu\delta^\mu_\alpha=-j_\alpha \end{align*}
また、Euler-Lagrange方程式の後半部分の計算を進めましょう。残る添え字が$\alpha$,$\beta$になるように進めましょう。
\begin{align*} \dfrac{\partial}{\partial(\partial_\beta A^\alpha)}\left(F_{\mu\nu}F^{\mu\nu}\right) &=\dfrac{\partial}{\partial(\partial_\beta A^\alpha)}\left\{\left(\partial_\mu A_\nu-\partial_\nu A_\mu\right)\left(\partial^\mu A^\nu-\partial^\nu A^\mu\right)\right\} \\ &=\left\{\dfrac{\partial}{\partial(\partial_\beta A^\alpha)}\left(\eta_{\nu\rho}\partial_\mu A^\rho-\eta_{\mu\rho}\partial_\nu A^\rho\right)\right\}\left(\partial^\mu A^\nu-\partial^\nu A^\mu\right) +\left(\partial_\mu A_\nu-\partial_\nu A_\mu\right)\dfrac{\partial}{\partial(\partial_\beta A^\alpha)}\left(\eta^{\mu\rho}\partial_\rho A^\nu-\eta^{\nu\rho}\partial_\rho A^\mu\right) \\ &=(\eta_{\nu\rho}\delta^\beta_\mu\delta^\rho_\alpha-\eta_{\mu\rho}\delta^\beta_\nu\delta^\rho_\alpha)(\partial^\mu A^\nu-\partial^\nu A^\mu) +(\partial_\mu A_\nu-\partial_\nu A_\mu)(\eta^{\mu\rho}\delta^\beta_\rho\delta^\nu_\alpha-\eta^{\nu\rho}\delta^\beta_\rho\delta^\mu_\alpha) \\ &=\eta_{\nu\rho}\delta^\beta_\mu\delta^\rho_\alpha(\partial^\mu A^\nu-\partial^\nu A^\mu)-\eta_{\mu\rho}\delta^\beta_\nu\delta^\rho_\alpha(\partial^\mu A^\nu-\partial^\nu A^\mu) +(\partial_\mu A_\nu-\partial_\nu A_\mu)\eta^{\mu\rho}\delta^\beta_\rho\delta^\nu_\alpha-(\partial_\mu A_\nu-\partial_\nu A_\mu)\eta^{\nu\rho}\delta^\beta_\rho\delta^\mu_\alpha \\ &=(\partial^\beta A_\alpha-\partial_\alpha A^\beta)-(\partial_\alpha A^\beta-\partial^\beta A_\alpha)+(\partial^\beta A_\alpha-\partial_\alpha A^\beta)-(\partial_\alpha A^\beta-\partial_\beta A^\alpha) \\ &=4(\partial^\beta A_\alpha-\partial_\alpha A^\beta) \\ &=4\eta_{\alpha\rho}(\partial^\beta A^\rho-\partial^\rho A^\beta) \\ &=4\eta_{\alpha\rho}F^{\beta\rho} \\ &=4F^\beta_{\ \ \ \alpha} \end{align*}
というわけで、係数もつけてEuler-Lagrange方程式に代入すれば、
\begin{align*} -j_\alpha+\dfrac{1}{\mu}\partial_\beta F^\beta_{\ \ \ \alpha}=0 \end{align*}
添え字をあげて、整理して、ダミー変数をつけかえれば、
\begin{align*} \partial_\mu F^{\mu\nu}=\mu j^\nu \end{align*}
となります。

ラグランジアンの係数

ラグランジアンの係数がなぜ$-1/4$なのか?それは、ハミルトニアンが
\begin{align*} H=\dfrac{1}{2}\varepsilon\|\boldsymbol{E}\|^2+\dfrac{1}{2\mu}\|\boldsymbol{B}\|^2 \end{align*}
と表されたことによります。



このエントリーをはてなブックマークに追加